Đến nội dung

Hình ảnh

$\frac{x^3}{y^3}+\frac{y^3}{z^3}+\frac{z^3}{x^3} \geqslant $


  • Please log in to reply
Chủ đề này có 4 trả lời

#1
Night and Fury

Night and Fury

    Lính mới

  • Thành viên
  • 7 Bài viết

Cho x,y,y >0 

CMR

$\frac{x^3}{y^3}+\frac{y^3}{z^3}+\frac{z^3}{x^3} \geqslant  \frac{x^2}{y^2}+\frac{y^2}{z^2}+\frac{z^2}{x^2}$


Bài viết đã được chỉnh sửa nội dung bởi Night and Fury: 18-01-2015 - 19:56


#2
dogsteven

dogsteven

    Đại úy

  • Thành viên
  • 1567 Bài viết

$a=\dfrac{x}{y}, b=\dfrac{y}{z}, c=\dfrac{z}{x}\Rightarrow abc=1$

Ta cần chứng minh $a^3+b^3+c^3\geqslant a^2+b^+c^2$

$3(a^3+b^3+c^3)\geqslant (a+b+c)(a^2+b^2+c^2)\geqslant 3(a^2+b^2+c^2)$


Quyết tâm off dài dài cày hình, số, tổ, rời rạc.


#3
kanashini

kanashini

    Trung sĩ

  • Thành viên
  • 139 Bài viết
$\dfrac{x^3}{y^3}+\dfrac{x^3}{y^3}+1 \ge 3.\dfrac{x^2}{y^2}$
 
$\dfrac{y^3}{z^3}+\dfrac{y^3}{z^3}+1 \ge 3.\dfrac{y^2}{z^2}$
 
$\dfrac{z^3}{x^3}+\dfrac{z^3}{x^3}+1 \ge 3.\dfrac{z^2}{x^2}$
 
Cộng theo vế,được:
 
$2(\dfrac{x^3}{y^3}+\dfrac{y^3}{z^3}+\dfrac{z^3}{x^3})+3 \ge 3(\dfrac{x^2}{y^2}+\dfrac{y^2}{z^2}+\dfrac{z^2}{x^2}) $
 
$\Leftrightarrow 2(\dfrac{x^3}{y^3}+\dfrac{y^3}{z^3}+\dfrac{z^3}{x^3}) \ge 2(\dfrac{x^2}{y^2}+\dfrac{y^2}{z^2}+\dfrac{z^2}{x^2})+(\dfrac{x^2}{y^2}+\dfrac{y^2}{z^2}+\dfrac{z^2}{x^2}-3) \ge 2(\dfrac{x^2}{y^2}+\dfrac{y^2}{z^2}+\dfrac{z^2}{x^2})$
 
Do $\dfrac{x^2}{y^2}+\dfrac{y^2}{z^2}+\dfrac{z^2}{x^2} \ge 3$ (Cau-chy)
 
$\Rightarrow$ đpcm


#4
huykinhcan99

huykinhcan99

    Sĩ quan

  • Điều hành viên OLYMPIC
  • 336 Bài viết

Cho x,y,y >0 

CMR

$\frac{x^3}{y^3}+\frac{y^3}{z^3}+\frac{z^3}{x^3} \geqslant  \frac{x^2}{y^2}+\frac{y^2}{z^2}+\frac{z^2}{x^2}$

 

Áp dụng bất đẳng thức $AM - GM$ cho $9$ số ta có:
$$\underbrace{\dfrac{x^3}{y^3}+\dfrac{x^3}{y^3}+\ldots+\dfrac{x^3}{y^3}}_{\text{7 số}} +\dfrac{y^3}{z^3}+\dfrac{z^3}{x^3} \geqslant 9 \sqrt[9]{\dfrac{x^{21}.y^3.z^3}{y^{21}.z^3.x^3}}$$
\begin{equation} \label{eq:1} \Leftrightarrow 7\dfrac{x^3}{y^3} +\dfrac{y^3}{z^3} +\dfrac{z^3}{x^3} \geqslant 9\dfrac{x^2}{y^2} \end{equation}
 
Tương tự ta có:
\begin{align} \label{eq:2} 7\dfrac{y^3}{z^3} +\dfrac{z^3}{x^3} +\dfrac{x^3}{y^3} \geqslant 9\dfrac{y^2}{z^2} \\ \label{eq:3} 7\dfrac{z^3}{x^3} +\dfrac{x^3}{y^3} +\dfrac{y^3}{z^3} \geqslant 9\dfrac{z^2}{x^2} \end{align}
 
Cộng ba bất đẳng thức \eqref{eq:1}, \eqref{eq:2}, \eqref{eq:3} vế với vế ta thu được 
$$9\left( \dfrac{x^3}{y^3}+\dfrac{y^3}{z^3}+\dfrac{z^3}{x^3} \right) \geqslant 9\left( \dfrac{x^2}{y^2}+\dfrac{y^2}{z^2}+\dfrac{z^2}{x^2}\right)$$
\begin{equation} \tag{$\blacksquare$} \Leftrightarrow \dfrac{x^3}{y^3}+\dfrac{y^3}{z^3}+\dfrac{z^3}{x^3} \geqslant \dfrac{x^2}{y^2}+\dfrac{y^2}{z^2}+\dfrac{z^2}{x^2} \end{equation}
 
Dấu bằng xảy ra khi $x=y=z$

Bài viết đã được chỉnh sửa nội dung bởi huykinhcan99: 18-01-2015 - 21:48

$$\text{Vuong Lam Huy}$$

#5
huykinhcan99

huykinhcan99

    Sĩ quan

  • Điều hành viên OLYMPIC
  • 336 Bài viết

Hoặc có thể $AM - GM$ như sau:

 

Áp dụng bất đẳng thức $AM - GM$ cho $8$ số ta có:
$$\underbrace{\dfrac{x^3}{y^3}+\dfrac{x^3}{y^3}+\ldots+\dfrac{x^3}{y^3}}_{\text{6 số}} +\dfrac{y^2}{z^2}+\dfrac{z^2}{x^2} \geqslant 8 \sqrt[8]{\dfrac{x^{18}.y^2.z^2}{y^{18}.z^2.x^2}}$$
\begin{equation} \label{1eq:1} \Leftrightarrow 6\dfrac{x^3}{y^3} +\dfrac{y^2}{z^2} +\dfrac{z^2}{x^2} \geqslant 8\dfrac{x^2}{y^2} \end{equation}
 
Tương tự ta có:
\begin{align} \label{1eq:2} 6\dfrac{y^3}{z^3} +\dfrac{z^2}{x^2} +\dfrac{x^2}{y^2} \geqslant 8\dfrac{y^2}{z^2} \\ \label{1eq:3} 6\dfrac{z^3}{x^3} +\dfrac{x^2}{y^2} +\dfrac{y^2}{z^2} \geqslant 8\dfrac{z^2}{x^2} \end{align}
 
Cộng ba bất đẳng thức \eqref{1eq:1}, \eqref{1eq:2}, \eqref{1eq:3} vế với vế ta thu được 
$$6\left( \dfrac{x^3}{y^3}+\dfrac{y^3}{z^3}+\dfrac{z^3}{x^3} \right) +2\left( \dfrac{x^2}{y^2}+\dfrac{y^2}{z^2}+\dfrac{z^2}{x^2}\right) \geqslant 8\left( \dfrac{x^2}{y^2}+\dfrac{y^2}{z^2}+\dfrac{z^2}{x^2}\right)$$
$$\Leftrightarrow 6\left( \dfrac{x^3}{y^3}+\dfrac{y^3}{z^3}+\dfrac{z^3}{x^3} \right) \geqslant 6\left( \dfrac{x^2}{y^2}+\dfrac{y^2}{z^2}+\dfrac{z^2}{x^2}\right)$$
\begin{equation} \tag{$\blacksquare$} \Leftrightarrow \dfrac{x^3}{y^3}+\dfrac{y^3}{z^3}+\dfrac{z^3}{x^3} \geqslant \dfrac{x^2}{y^2}+\dfrac{y^2}{z^2}+\dfrac{z^2}{x^2} \end{equation}
 
Dấu bằng xảy ra khi $x=y=z$

Bài viết đã được chỉnh sửa nội dung bởi huykinhcan99: 18-01-2015 - 21:48

$$\text{Vuong Lam Huy}$$




1 người đang xem chủ đề

0 thành viên, 1 khách, 0 thành viên ẩn danh